You are on page 1of 17

EC-Gate 2016 SET-3

Page 1

ECe set-3 2016


Q1 An apple costs Rs. 10. An onion costs Rs.8. In the given set of number, all are perfect squares but 97
Select the most suitable sentence with respect to grammar is not
and usage. 324 is square of 18 & ^18h2 = 324
(A) The price of an apple is greater than an onion 441 is square of 21 & ^21h2 = 441
(B) The price of an apple is more than onion 64 is square of 8 & ^8 h2 = 64
(C) The price of an apple is greater than that of an on- 97 is not the square of any number
ion ` The number that least fits in gives set is 97.
(D) Apples are more costlier than onions
Q5 It takes 10s and 15s respectively, for two trains
S1 Correct option is (C) travelling at different constant speeds to completely pass
Based on the given sentences option ‘C’ is the correct a telegraph post. The length of the first train is 120 m
and that of the second train is 150 m. The magnitude of
sentence which is in the comparative degree. Option A
the difference in the speeds of the two trains (in m/s) is
and B convey the wrong comparison and D has double
______.
comparative and so they are wrong. (A) 2.0
(B) 10.0
Q2 The Buddha said, “Holding on to anger is like
(C) 12.0
grasping a hot coal with the intent of throwing it at
someone else; you are the one who gets burnt.” (D) 22.0
(A) Burning
(B) igniting S1 Correct option is (A)
(C) clutching Speed of the train (ST)
length of the train (LT) + Distance (D)
(D) flinging =
Time (T)
(ST) = LT + D
S1 Correct option is (C) T
The underlined word grasping means clutching or holding D = Distance (or) length of the platform = 0
something tightly. ` Speed of the first train ^ST1h
= 120
10
Q3 M has a son Q and a daughter R. He has no other = 12 m/s
children. E is the mother of P and daughter-in-law of M. Speed of the second train ^ST h
2
How is P related to M? 150
(A) P is the son-in-law of M =
15
(B) P is the grandchild of M = 10 m/s
(C) P is the daughter in law of M ` The magnitude of the difference in the speed of the
two trains (m/s)
(D) P is the grandfather of M
= 12 − 10
=2
S1 Correct option is (B)
Q and R are the son and daughter of M, E is the mother Q.6-Q.10 carry two marks each
of P and daughter-in-law of M means Q and E are married
couples in the family. Q6 The velocity V of a vehicle along a straight line
` P is the grandchild of M. measured in m/s and plotted as shown with respect to
time in seconds. At the end of the 7 seconds, how much
Q4 The number that least fits this set: (324, 441, 97, will the odometer reading increase by (in m)?
and 64) is ______
(A) 324
(B) 441
(C) 97
(D) 64

S1 Correct option is (C)


Page 2 EC-Gate 2016 SET-3

(D) Stray dogs are the main source of rabies worldwide.

S1 Correct option is (A)


Only option ‘A’ can be logically inferred from the
information provided in the argument.

Q8 A flat is shared by four first year undergraduate


students. They agreed to allow the oldest of them to enjoy
some extra space in the flat. Manu is two months older
than Sravan, who is three months younger than Trideep.
(A) 0 Pavan is one month older than Sravan. Who should
occupy the extra space in the flat?
(B) 3
(A) Manu
(C) 4
(B) Sravan
(D) 5
(C) Trideep
(D) pavan
S1 Correct option is (D)
The odometer reading increase from starting point to end
S1 Correct option is (C)
point
Manu age = Sravan age + 2 months
Area of the velocity and time graph per second
Manu age = Trideep age - 3 months
1 st sec & triangle = 1 # 1 # 1 Pavan age = Sravan’s age + 1 month
2
= 1 From this Trideep age>Manu>Pavan>Sravan
2 ` Trideep can occupy the extra spane in the flat.
nd
2 sec & square = 1 # 1 = 1

3 rd sec & square+triangle = 1 # 1 + 1 # 1 # 1 Q9 Find the area bounded by the lines 3x + 2y = 14 ,


2 2x − 3y = 5 in the first quadrant.
=1 1 (A) 14.95
2
th 1 (B) 15.25
4 sec & triangle = # 1 # 2
2 (C) 15.70
=1 (D) 20.35
th
5 sec & straight line = 0
th
6 sec & triangle = 1 # 1 # 1 S1 Correct option is (B)
2
= 1
2
7 th sec 1
& triangle = # 1 # 1
2
=1
2
Total Odometer reading at 7 seconds
= 1 + 1 + 11 + 1 + 0 + 1 + 1
2 2 2 2
=5

Q7 The overwhelming number of people infected with


rabies in India has been flagged by the World Health A = ;14 , 0E
3
Organization as a source of concern. It is estimated that
inoculating 70% of pets and stray dogs against rabies can B = 60, 7@
lead to a significant reduction in the number of people
infected with rabies. Which of the following can be C = :5 , 0D
2
logically inferred from the above sentences?
(A) The number of people in India infected with rabies D = :0, − 5D
3
E = 64, 1@
is high

(B) The number of people in other parts of the world F = 60, 1@
who are infected with rabies is low. Required area = Area of Tle OAB − Area of Tle CEA
(C) Rabies can be eradicated in India by vaccinating
70% of stray dogs. = b 1 # 14 # 7 l − ;1 # b 14 − 5 l # 1E

2 3 2 3 2
EC-Gate 2016 SET-3 Page 3

= 15.25 sq. units


3
Another method: = 1 − z + z − ......
Required area = Area of Tle BFE + Area of FEOC z 3! 5!
= 1 # 4 # 6 + 1 # ^4 + 2.5h # 1 Res. f ^z h = 1
2 2 z = 0
= 12 + 3.25
= 15.25 sq. units. Q13 The probability of getting a “head” in a single toss
of a biased coin is 0.3. The coin is tossed repeatedly till
Q10 A straight line is fit to a data set ( ln x, y ). This line a “head” is obtained. If the tosses are independent, then
intercepts the abscissa at ln x = 0.1 and has a slope of the probability of getting “head” for the first time in the
- 0.02 . What is the value of y at x = 5 from the fit? fifth toss is ______
(A) - 0.030
(B) - 0.014 S1 Correct answer is 0.07203
(C) 0.014 P = ]0.7g4 ^0.3h
= 0.07203
(D) 0.030
1
dx
S1 Correct option is (A) Q14 The integral #
0 ^ - xh
1
is equal to ______
Straight line equation y = mx + c
m = slope =− 0.02 S1
Correct answer is 2
dx = "− 2 1 − x ,0
1
set (log x, y ) #
^1 - x h
1

If log x = X , then set ^x, y h 6x = 0.1, y = 0@


=− 2 6^0 h − 1@
0

y = mX + C
=2
0 =− 0.02 # 0.1 + C
` C = 0.002
y = mX + C Q15 Consider the first order initial value problem
y =− 0.02 # log x + C yl = y + 2x − x2 ,
@x = 5 y ^0 h = 1, ^0 < x < 3h
y =− 0.02 # log 5 + 0.002 with exact solution y ^x h = x2 + ex . For x = 0.1 the
=− 0.030 percentage difference between the exact solution and the
solution obtained using a single iteration of the second-
Q.11-Q.35 Carry one mark each order Runge-Kutta method with step-size h = 0.1 is
______
Q11 Consider a 2 # 2 square matrix
σ x S1 Correct answer is 0.6
A = >
ω σH
dy
= y + 2x − x 2
dx
y ^0 h = 1, ^0 < x < 3h
Where x is unknown. If the eigen values of the matrix A
are ^σ + jωh and ^σ - jωh, then x is equal to

Given f ^x, y h = y + 2x − x2
(A) + jω
x 0 = 0
(B) - jω y 0 = 1
(C) + ω h = 0.1
(D) - ω k1 = hf ^x 0, y 0h
= 0.1 _1 + 2 ^0 h − ^0 h2i
S1 Correct option is (D) = 0.1
det ^Ah = σ2 − ωx k2 = hg ^x 0 + h, y 0 + k1h
= σ2 + ω2 = 0.1 ^y 0 + k1h + 2 ^x 0 + h h − ^x 0 + h h2
= σ2 − ωx = 0.1 ^1 + 0.1h + 2 ^0.1h − ^0.1h2
= ω2 = 0.1 ^1.1 + 0.2 − 0.01h
=− ωx = 0.129
y1 = y 0 + 1 ^k1 + k2h
2
σ + ωx = 0
`
` x =− ω 2
= 1 + ^0.1 + 0.129h
1
sin ^z h
2
Q12 For f ^z h = , the residue of the pole at z = 0 is = 1 + 0.1145
z2
______ = 1.1145
Exact solution, y ^x h = x2 + ex
S1 Correct answer is 1 y ^0.1h = ^0.1h2 + e0.1
sin ^z h
= 12 'z − z + z − ......1
3 2
= 0.01 + 1.1052

z 2
z 3! 5!
Page 4 EC-Gate 2016 SET-3

= 1.1152 (A) y 6n@ = x 6n@


ERROR = 1.1152 − 1.1145 (B) y 6n@ = x 6− n@
= 0.00062 (C) y 6n@ =− x 6n@

(D) y 6n@ =− x 6− n@
Percentage Error = 0.00062 # 100
0.06%

Q16 Consider the signal x ^ t h = cos ^6pt h + sin ^8pt h S1 Correct option is (C)
, where t is in seconds. The Nyquist sampling rate (in ^a hn x ^n h ) X c az m
samples/second) for the signal y ^ t h = x ^2t + 5h is
a =− 1
x ^n h = d 6n − 3@ + 2d 6n − 5@
(A) 8
but
(B) 12
y ^n h = ^− 1hn x ^n h
= ^− 1hn 8d ^n − 3h + 2d 6n − 5@
(C) 16

(D) 32
y ^n h =− d ^n − 3h − 2d ^n − 5h
S1
Correct option is (C) =− x ^n h
x ^ t h = cos ^6pt h + sin ^8pt h
y ^ t h = x ^2t + 5h Q19 In the RLC circuit shown in the figure, the input
y ^ t h = cos ^12pt + 30ph + sin ^16pt + 40ph voltage is given by vi ^ t h = 2 cos ^200t h + 4 sin ^500t h. The
fm1 = 6 output voltage v 0 ^ t h is
fm2 = 8
fm = 8 Hz
^ fs hmin = 2fm
= 16 Hz

If the signal x ^ t h = p^t h * sinp^t h with * denoting the


sin t t
Q17
convolution operation, then x ^ t h is equal to
sin ^ t h
(A)
πt
(A) cos ^200t h + 2 sin ^500t h
sin ^2t h
(B)
πt (B) 2 cos ^200t h + 4 sin ^500t h
2 sin ^ t h (C) sin 200t + 2 cos 500t
(C)
πt (D) 2 sin ^200t h + 4 cos ^500t h
sin ^ t h 2
(D) c
πt m S1 Correct option is (B)
Given
S1 Correct option is (A) Vi ^ t h = 2 cos ^200t h + 4 sin 500t
Let us apply SPT [Super Position Theorem] only consider
2 cos 200t , then circuit becomes

Convolution of two sinc pulses is sinc pulse.


sin ^ t h
x1 ^ t h =
πt
x ^ t h x1 ^ t h* x1 ^ t h
X ^ ω h = X 1 ^w h $ X 1 ^w h So, V0l^ t h = 2 cos 200t
= X 1 ^ω h Now only consider 4 sin 500t , then circuit becomes
x ^ t h = x 1 ^ t h
sin ^ t h
=

πt

Q18 A discrete-time signal x 6n@ = d 6n − 3@ + 2d 6n − 5@ has


z -transform X ^z h. If Y ^z h = X ^− z h is the z -transform of
another signal y 6n@, then
EC-Gate 2016 SET-3 Page 5

S1 Correct option is (A)


So, again V0ll^ t h = 4 sin 500t
Finally according to SPT Q22 The figure shows the I - V characteristic of a solar
V0 ^ t h = V0l^ t h + V0ll^ t h cell illuminated uniformly with solar light of power 100
V0 ^ t h = 2 cos ^200t h + 4 sin ^500t h mW/cm2 . The solar cell has an area of 3 cm2 and a fill
factor of 0.7. The maximum efficiency (in%) of the device
Q20 The I - V characteristics of three types of diodes at is ______
the room temperature, made of semiconductors X, Y and
Z are shown in the figure. Assume that the diodes are
uniformly doped and identical in all respects except their
materials. If EgX , EgY and EgZ are the band gaps of X, Y
and Z respectively, then
(A) EgX > EgY > EgZ
(B) EgX = EgY = EgZ
(C) EgX < EgY < EgZ
(D) no relationship among these band gaps exists S1 Correct answer is 21
Fill factor = 0.7

= Pmax
PT
= Pmax
ISC $ VOC
" Pmax = 63 # 10−3 W
(PT " Theoretical power)
η max = Pmax
Pin
= 63 # 10−3 W
2 # 100
S1 Correct option is (C) 100 # 10−3 cm
W
# 3 cm
2

Where Vr is cut in voltage


= 21%

Vr > Vr > Vr
3 2 1

Vr \ Eg
So, EgZ > EgY > EgX Q23 The diodes D1 and D2 in the figure are ideal and
the capacitors are identical. The product RC is very large
compared to the time period of the ac voltage. Assuming
that the diodes do not breakdown in the reverse bias, the
output voltage V0 (in volt) at the steady state is ______

Q21 The figure shows the band diagram of a Metal Oxide


Semiconductor (MOS). The surface region of this MOS is
in
(A) inversion
(B) accumulation S1 Correct answer is 0V
(C) depletion
(D) flat band
Page 6 EC-Gate 2016 SET-3

Diodes are ideal therefore during Positive cycle of input


V0 = 10 − 10
= 0V
During Negative cycle, the diodes are Reverse biased
V0 = 0V
` V0 = 0 V (always)

Q24 Consider the circuit shown in the figure. Assuming


VBE1 = VEB2 = 0.7 volt, value of the dc voltage VC2 (in volt) S1 Correct answer is 5.65
is ______

T1 = 0.693 ^RA + RB h C
= 0.693 ^2.2 k + 4.7 kh 0.022µ
= 0.1052 msec
S1
Correct answer is 0.5 V Frequency of oscillations ^ f h
VE1 = 2.5 − 0.7
= 1
= 1.8 V T
VB2 = VE1 − VEB2 = 5.65 kHz
= 1.8 − 0.7
= 1.1 V Q26 In an 8085 microprocessor, the contents of the
IB2 = VB2 − 1 accumulator and the carry flag are A7 (in hex) and 0,
10 k
respectively. If the instruction RLC is executed then the
= 1−1
1 .
contents of the accumulator (in hex) and the carry flag,
10 k
respectively, will be
= 0.1 (A) 4E and 0
10 k
IC2 = βIB2 (B) 4E and 1
(C) 4F and 0
= 50 ; 0.1 E (D) 4F and 1
10 k
VC2 = IC2 ^1 kh
50 ^0.1h S1 Correct option is (D)
10 k ^ h
=
1k Given A = A7 H = 1 0 1 0 0 1 1 0
CY

= 0.5 V
After executing RLC
A = 1 0 0 0 1 1 1 1
Q25 In a stable multivibrator circuit shown in the figure,
A = 4FH and cy = 1
the frequency of oscillation (in kHz) at the output pin 3
is ______ Q27 The logic functionality realized by the circuit shown
below is
EC-Gate 2016 SET-3 Page 7

figure, the open-loop transfer function G ^s h is given as


G ^s h = s^s 2+ 1h . The steady state error ess due to a unit step
input is

(A) OR
(B) XOR
(C) ANAD (A) 0
(D) AND (B) 0.5
(C) 1.0
S1 Correct option is (D) (D) 3
It is a AND gate
A B Y S1 Correct option is (A)
0 0 0 G ^ s h = 2 ,
s ^s + 1h
Given
0 1 0
H ^ s h = 1
1 0 0
& Type - 1 system, to the unit step input the ess = 0
1 1 1
Q31 For a superheterodyne receiver, the intermediate
Q28 The minimum number of 2-input NAND gates frequency is 15 MHz and the local oscillator frequency
required to implement a 2-input XOR gate is is 3.5 GHz. If the frequency of the received signal is
(A) 4 greater than the local oscillator frequency, then the image
(B) 5 frequency (in MHz) is _____
(C) 6
S1
Correct answer is 3485 MHz
(D) 7 fIf = 15 MHz
FLo = 3500 MHz
S1 Correct option is (A) fs - fLo = fIf
Minimum number of NAND gates required for 2-input fs = fLo + fIf
EX-OR gate=4 = 3515 MHz

Q29 The block diagram of a feedback control system is


shown in the figure. The overall closed-loop gain G of the
system is

fsi = image frequency


= fs − 2fIf
= 3515 − 2 # 15
= 3485 MHz

(A) G = G1 G 2 Q32 An analog baseband signal, bandlimited to 100 Hz,


1 + G1 H1 is sampled at the Nyquist rate. The samples are quantized
(B) G = G1 G 2 into four message symbols that occur independent with
1 + G1 G 2 + G1 H1
probabilities p1 = p 4 = 0.125 and p2 = p 3 . The information
(C) G = G1 G 2 rate (bits/sec) of the message source is ______
1 + G 1 G 2 H1
(D) G = G1 G 2 S1
Correct answer is 362.255
1 + G 1 G 2 + G 1 G 2 H1
P1 = 0.125
S1 Correct option is (B) P4 = 0.125
From block diagram P2 = 0.375
Y ^s h P3 = 0.375
= G ^ s h
X ^s h

G1 G 2 H = 0.125 log 2 1 + 0.125 log 2 1
= 0.125 0.125
1 + G1 H1 + G1 G 2
+ 0.375 log 2 1 + 0.375 log 2 1
0.375 0.375
Q30 For the unity feedback control system shown in the = 1.811
Page 8 EC-Gate 2016 SET-3

Information rate (LHCP)


= H # 200
= 362.255 Q35 Faraday’s law of electromagnetic induction is
mathematically described by which one of the following
Q33 A binary baseband digital communication system equations?
1
0 < t < TS (A) d:Bv = 0
employs the signal p ^ t h = T S
.
0 otherwise (B) d:Dv = ρv
For transmission of bits. The graphical representation of
the matched filter output y ^ t h for this signal will be (C) d # Ev =−2B
v
2t
v
(D) d # Hv = σEv + 2D
2t

S1 Correct option is (C)


Differential form of Faraday’s law is given by
v
d # Ev =−2B or
2t
v
d # Ev =− µ2H
2t
Q.36-Q.55 carry two marks each

Q36 The particular solution of the initial value problem


S1 Correct option is (C) given below is
+ 12 + 36y = 0 with y ^0 h = 3 and
d2 y dy

dx2 dx
dy
=− 36
dx x = 0
(A) ^3 - 18x h e-6x
(B) ^3 + 25x h e−6x
(C) ^3 + 20x h e−6x
Q34 If a right-handed circularly polarized wave is incident (D) ^3 - 12x h e-6x
normally on a plane perfect conductor, then the reflected
wave will be S1 Correct option is (A)
(A) right-handed circularly polarized D2 + 12D + 36 = 0
(B) left-handed circularly polarized & =− 6, − 6
(C) elliptically polarized with a tilt angle of 45c The solution is y = C1 e−6x + C 2 xe−6x ...(1)
(D) horizontally polarized y ^ 0 h = 3 & 3 = C 1
(1) & y = e−6x + C 2 xe−6x
=− 18e−6x + C 2 "− 6xe−6x + e−6x ,
S1 Correct option is (B) dy

dx
dy
& =− 18 + C2
dx x = 0
& - 36 =− 18 + C2
& C2 =− 18
` The solution is y = 3e−6x − 18xe−6x

Q37 If the vectors e1 = ^1, 0, 2h, e2 = ^0, 1, 0h and


e 3 = ^− 2, 0, 1h form an orthogonal basis of the three
If the wave is incident on perfect conductor then reflection dimensional real space R3 , then the vector u = ^4, 3, − 3h εR3
coefficient is given by can be expressed as
Er
Γ / (A) u =− 2 e1 − 3e2 − 11 e 3
0

Ei
0 5 5
=− 1
(B) u =− 2 e1 − 3e2 + 11 e 3
Er = Ei +180c
0 0
5 5
If incident wave is traveling along + Z direction then the
(C) u =− 2 e1 + 3e2 + 11 e 3
reflected wave will be traveling along - Z direction. 5 5
` The reflected wave is left hand circularly polarized (D) u =− 2 e1 + 3e2 − 11 e 3
5 5
EC-Gate 2016 SET-3 Page 9

S1 Correct option is (D) H ^ jω 0h = 2 cos b 2π l


u = x 1 e 1 + x 2 e 2 + x 3 e 3 3
^4, 3, - 3h = x1 ^1, 0, 2h + x2 ^0, 1, 0h + x 3 ^− 2, 0, 1h = 2 b − 1 l =− 1
2
x1 - 2x 3 = 4 ...(3)
On solving these equations, we get y ^ t h = 2 cos b 2π t + 180cl
3
x1 =− 2 ,
5
x ^ t h = cos πt
ω 0 = π
H ^ jω 0h = 2 cos ^πh
x 2 = 3 ,

x 3 = − 11 =− 2
5
y ^ t h = 2 cos ^πt + 180ch
` u =− 2e1 + 3e2 − 11 e 3
5 5
y ^ t h = 2 cos b 2p t + p l − 2 cos ^pt + ph
3
Q38 A triangle in the xy -plane is bounded by the straight
lines 2x = 3y , y = 0 and x = 3 . The volume above the ω 1 = 2π
3
triangle and under the plane x + y + z = 6 is ______ ω 2 = π
T1 = 3
S1 Correct answer is 10 T2 = 2
2x

^6 − x − y h dxdy
3 T0 = 6
Volume zdxdy = ## # #
3

x=0 y=0 ω 0 = 2π
= 10 T0

3
# ^ h cos ^2ω 0 t + πh − 2 cos ^2ω 0 t + πh
z
Q39 The values of the integral 21πj e
z-2 dz along a closed y t = 2
contour c in anti-clockwise direction for
y ^ t h = e j^2ω t + πh + e−j^2ω t + πh − e j^3ω t + πh − e−j^3ω t + πh
c

(i) the point z 0 = 2 inside the contour c , and 0 0 0 0

y ^ t h =− e j^ h − e−j^2w t h + e j^3w t h + e−j^3w t h


2w t
(ii) the point z 0 = 2 outside the contour c , respectively,
0 0 0 0

are C 3 = 1
(A) (i) 2.72 (ii) 0
(B) (i) 7.39 (ii) 0 Q41 The ROC (region of convergence) of the z -transform
of a discrete-time signal is represented by the shaded region
is the z -plane. If the signal x 6n@ = ^2.0h n , − 3 < n < + 3
(C) (i) 0 (ii) 2.72
(D) (i) 0 (ii) 7.39 then the ROC of its z -transform is represented by

S1 Correct option is (B)


(i) 1 #
2πj ^z - 2h 2pj ^ h
ez dz = 1 2pjf 2
c
& e2 = 7.39
(ii) 1 ez dz = 0 (a z = 2 lies outside c )
#
2πj ^z - 2h
c

Q40 A signal 2 cos ^ 23π t h - cos ^πt h is the input to an LTI


system with the transfer function H ^s h = es + e−s . If Ck
denotes the k th coefficient in the exponential Fourier series
of the output signal, then C 3 is equal to
(A) 0
(B) 1
(C) 2
(D) 3

S1
Correct option is (B)
H ^e jω h = e jw + e−jw
= 2 cos ω

It x ^ t h = 2 cos b 2π t l
3
ω 0 = 2π
3
Page 10 EC-Gate 2016 SET-3

______

S1 Correct option is (D)


x ^y h = ^2 hn u ^n h + b 1 l u ^− n − 1h
n

2
roc = ^ z > 2h k _ z < 12 i S1 Correct answer is i =− 1A

No ROC

Q42 Assume that the circuit in the figure has reached


the steady state before time t = 0 when the 3Ω resistor
suddenly burns out, resulting in an open circuit. The
current i ^ t h (in ampere) at t = 0+ is ______

Nodal
^V − 8h V ^V − 8h V
+ + + = 0
1 1 1 1
4V = 16
S1 Correct answer is 0.98 to 1.02 V = 4 Volts
Here direction of current & correct component was not Now KCL
mentioned i − 4 + 4 + 1 = 0
Circuit at t = 0 − i =− 1A

z11 z12
The z -parameter matrix >
z21 z22H
Q44 for the two-port
network shown is

Now t = 0+
2 -2
(A) >
-2 2 H
2 2
(B) > H
2 2
9 -3
(C) >
6 9H
9 3
(D) > H
6 9
So, i ]0 +g = − 4
4
S1 Correct option is (A)
=− 1A
This is in Lattice form
` The magnitude of the current is 1 Amp
where Za = 3Ω
Zb = 0Ω
Q43 In the figure shown, the current i (in ampere is Zc = 0Ω
EC-Gate 2016 SET-3 Page 11

Zd = 6Ω S1 Correct answer is 8
But it is not symmetrical & balanced The number of complex multiplications required for DIF-
FFT
= b N log 2 N l
2
` b 2 log 2 N l^20 µ sech = 125 µ sec
N

Q46 The direct form structure of an FIR (finite impulse


response) filter is shown in the figure.

The filter can be used to approximate a


So, Z11 = V1 (A) low-pass filter
I1 I2 = 0
(B) high-pass filter
= 3//6 = 2 Ω
(C) band-pass filter
Z21 = V2 (D) band-stop filter
I1 I2 = 0

S1 Correct option is (C)


y ^n h = 5 6x ^n h − x ^n − 2h@
Y ^e jω h = 5 81 − e−2jw X ^e jw hB
H ^e jω h = 5 61 − e−2jω

KVL
− V2 − 2I1 + 0 = 0
- V2 =− 2I1
& Z21 =− 2Ω
Also
So, Z22 = V2 ω H ^e jω h
I2 I = 0 1

= 3//6 = 2Ω
0 0
π 10
Z21 = V1 2
I2 I = 0
KVL 1

π 0
- V1 - 2I2 = 0
V1 =− 2I1 So it is Band pass filter
& Z21 =− 2Ω
& Z12 =− 2Ω Q47 The injected excess electron concentration profile
So, final answer in the base region of an npn BJT, biased in the active
2 −2
Z = >
−2 2 H
region, is linear, as shown in the figure. If the area of the

emitter-base junction is 0.001 cm2 , µ n = 800 cm2 /(V-s) in
the base region and depletion layer widths are negligible,
Q45 A continuous-time speech signal xa ^ t h is sampled at then the collector current I (in mA) at room temperature
c
a rate of 8 kHz and the samples are subsequently grouped is ______
in blocks, each of size N . The DFT of each block is to be (Given: thermal voltage VT = 26 mV at room temperature,
computed in real time using the radix-2 decimation-in- electronic charge q = 1.6 # 10−19 C )
frequency FFT algorithm. If the processor performs all
operations sequentially, and takes 20 µs for computing
each complex multiplication (including multiplications by
1 and - 1) and the time required for addition/subtraction
is negligible, then the maximum value of N is ______
Page 12 EC-Gate 2016 SET-3

S1 Correct answer is 6.656 MA


(A) D1 only
IC = AeDn dn
dx (B) D2 only
(C) Both D1 & D2
= Aeµ n dn
dx (D) None of D1 & D2
IC = 0.001 # 1.6 # 10−19 # 800 # 0.026
#d
10 4 - 0 n
S1 Correct option is (A)
0.5 # 10-4
IC = 6.656 mA

Q48 Figures I and II show two MOS capacitors of unit


area. The capacitor in Figure I has insulator materials
X (of thickness t1 = 1 nm and dielectric constant ε 1 = 4
) and Y (of thickness t2 = 3 nm and dielectric constant
ε 2 = 20 ). The capacitor in Figure II has only insulator
material X of thickness tEq. . If the capacitors are of equal
capacitors, then the value of tEq (in nm) is ______

Here both zener diodes are in RB


VBZ1 = 80 V
VBZ2 = 70 V
D1 have list saturation current
When we will vary the voltage above 80 V
D1 get breaks down and will replaced by 80 V and through
it ‘3’ current can flow through it. But because of D2
we will take minimum current i.e. net current equals to
S1 Correct answer is 1.55 to 1.65 reverse saturation current of D2 as we know
C = εA
d
C = C 1 C 2
C1 + C 2
4
# 3 #2010
=> H 8.8521 # 10−12
1 # 10−9 −9

+ 3 #2010 4
1 # 10−9 −9

= 2.5 # 109 ε 0
C = e r e 0
teq
er e 0 The diode have least saturation will break down first and
teq = it will replaced by its break down voltage and the net
2.5 # 109 e 0
current equal upto other diode reverse saturation current.
= 4 # e 09

2.5 # 10 e 0
Q50 For the circuit shown in the figure, R1 = R2 = R 3 = 1Ω
, L = 1µH and C = 1µF . If the input Vin = cos ^106 t h, then
= 1.6 # 10−9 m

the overall voltage gain _VV i of the circuit is ______
= 1.6 m
out
in

Q49 The I - V characteristics of the zener diodes D1


and D2 are shown in Figure I. These diodes are used
in the circuit given in Figure II. If the supply voltage is
varied from 0 to 100 V. then breakdown occurs in
EC-Gate 2016 SET-3 Page 13

S1 Correct option is (C)

S1 Correct answer is -1
m1 ; 01 b gm2 02 l 03E
V0 = g r || 1 || r || r
Vin

Q52 In the circuit shown in the figure,


transistor M1 is in saturation and has transconductance
gm = 0.01 seimens. Ignoring internal parasitic capacitances
and assuming the channel length modulation λ to be zero,
the small signal input pole frequency (in kHz)is ______

Vx = 1 + 1
Vin 10−6 s
6 6
= 1 + 10 + s + 10
s s
V0 = < − 110 F $ Vx 6
1+ s

= V0

Vx
= −s 6

s + 10 S1 Correct answer is 57.8745 kHz
` V0 = V0 $ Vx
Vin Vx Vin
= :s + 106 D: s D
−s s + 106

=− 1
` V0 =− 1
Vin

Q51 In the circuit shown in figure, the channel length


modulation of all transistor is non-zero ^λ ! 0h. Also all
transistors operate in saturation and have negligible body
effect. The ac small signal voltage gain _VV i of the circuit
0 CM1 = 50 PF 61 − AV @
AV =− gm RD
in

is
=− 0.01 # 1
AV =− 10
CM1 = 50PF 61 + 10@
= 0.55 # 10−9 F
= 0.55 nF
fp = 1
2πRi CM1
fp = 1
2π # 5K # 0.55 # 10−9
= 57.8745 kHz

(A) - gm1 ^r01 //r02 //r03h


Q53 Following is the K-map of a Boolean function of
(B) - gm1 br0 // 1 //r03 l five variables, P, Q, R, S and X . The minimum sum-of-
gm3
produce (SOP) expression for the function is
(C) - gm1 ;r01 // b 1 //r02 l //r03E
gm2
(D) - gm1 ;r01 // b 1 //r03 l //r02E
gm3
Page 14 EC-Gate 2016 SET-3

If the clock (C1k) frequency is 1Ghz, the counter behaves


as a
(A) mod-5 counter
(B) mod-6 counter
(C) mod-7 counter
(D) mod-8 counter

(A) P QSX + P QS X + QR S X + QRS X S1 Correct option is (D)


th
(B) QSX + QS X At 6 C1k pulse (i.e. at 6ns)& Q2 Q1 Q 0 = 110 & It makes
NAND gate output ‘0’ at 8ns due to its delay. By that
(C) QSX + QS X
time counter receives 7 th, 8 th C1k pulse and counts 111,
(D) QS + QS
000. Thus it is a mod - 8 counter.

S1 Correct option is (B)


Q56 The first two rows in the Routh table for the
It is a 5-variable K-map
characteristic equation of a certain closed-loop control
= Q S X + QS X
system are given as

Q54 For the circuit shown in the figure, the delays of NOR
gates, multiplexers and inverters are 2 ns, 1.5 ns and 1 ns,
respectively. If all the inputs P, Q, R, S and T are applied
at the same time instant, the maximum propagation delay
(in ns) of the circuit is ______
The range of K for which the system is stable is
(A) − 2.0 < K < − 0.5
(B) 0 < K < 0.5
(C) 0 < K < 3
(D) 0.5 < K < 3

S1
Correct option is (D)
s3 1 2k + 3
s2 2k 4
for stability
s1
4
s0
S1 Correct answer is 6
T = 0 " NOR " MUXI " MUX2 4k2 + 6k − 4 > 0
2ns 1.5ns 1.5ns

` Delay = 2 + 1.5 + 1.5 k > − 2 >, k > 0.5


= 5ns 0.5 < k < 3
T = 1 " NOT " MUXI " NOR " MUX2
1ns 1.5ns 2ns 1.5ns
Q57 A second-order linear time-invariant system is
= 1 + 1.5 + 2 + 1.5
` Delay
described by the following state equations
= 6ns
dt 1 ^ h 1^ h ^ h
d x t + 2x t = 3u t
` Maximum delay = 6 n sec
2^ h 2^ h ^ h
d x t + x t = u t
Q55 For the circuit shown in the figure, the delay of the dt
bubbled NAND gate is 2ns and that of the counter is where x1 ^ t h and x2 ^ t h are the two state variables and
assumed to be zero u ^ t h denotes the input. If the output c ^ t h = x1 ^ t h, then
the system is
(A) controlled but not observable
(B) observable but not controllable
(C) both controllable and observable
(D) neither controllable nor observable

S1 Correct option is (A)


xo1 =− 2x1 + 3U
xo2 =− x2 + U
EC-Gate 2016 SET-3 Page 15

c = x 1 = R y ^τ h

= E 7y ^ t h Y ^t + τ hA
xo1 − 2 0 x1 1
> o H = > 0 − 1H>x H + >1H U
x2
Ry ^τ h = E 8X ^ t h − X ^t − T0h6X ^t + t h − X ^t + t − T0hB
2

x1
6c @ = 81 0B>x2H Ry ^τ h = E 6X ^ t h X ^t + t h − X ^ t hX ^t + t − T0h − X ^t − T0h
By applying Gilbert’s test, the system is controlled but X ^t + t h + X ^t − T0h X ^t + t − T0hA
not observable.
Ry ^τ h = 8RX ^t h − RX ^t − T0h − RX ^t + T0h + RX ^t hB
Q58 The forward-path transfer function and the feedback- Ry ^τ h = 2RX ^t h − RX ^t − T0h − RX ^t + T0h
path transfer function of a single loop negative feedback
control system are given as G ^s h = sK+^ 2s +h2 and H ^s h = 1
s+2
2

respectively. If the variable parameter K is real positive,


then the location of the breakaway point on the root locus
diagram of the system is ______

S1 Correct answer is - 3.41


K ^s + 2h
G ^ s h =
Q60 A voice-grade AWGN (additive white Gaussian
Given 2 ,
s + 2s + 2 noise) telephone channel has a bandwidth of 4.0 kHz and
H ^ s h = 1 two-sided noise power spectral density η2 = 2.5 # 10−5
dk = 0 Watt per Hz. If information at the rate of 52 kbps is to
Break away point& be transmitted over this channel with arbitrarily small bit
ds
d d s + 2 n = 0 error rate, then the minimum bit energy Eb (in mJ/bit)

ds s2 + 2s + 2 necessary is ______
1 ^s2 + 2s + 2h − ^s + 2h^2s + 2h
& > H = 0 S1 Correct answer is 31.503
^s2 + 2s + 2h
2
C = 52 kbps
B = 4 kHz
No = 2.5 10−5
2 #
N = 4 # 103 # 2.5 # 10−5 # 2

C = B log 2 :1 + S D
N
S = 1638.2
& - s2 - 4s - 2 = 0
& - 0.58 , - 3.41 Eb = S
Rb
Valid BAP is - 3.41
J/ sec
=
bits/ sec
Q59 A wide sense stationary random process X ^ t h = 31.503
= log 1 b1 + S l
passes through the LTI system shown in the figure. If C
the autocorrelation function of X ^ t h is RX ^τ h, then the

B N
autocorrelation function RY ^τ h of the output y ^ t h is equal & log 2 b1 + l =
S C
to N B
& b1 + N l = 2C/B
S

= 213
= 8192
& = S
N
= 8191
(A) 2RX ^t h + RX ^t − T0h + RX ^t + T0h & S = 8191 # 4 # 103 # 2.5 # 10−5 # 2
(B) 2RX ^t h − RX ^t − T0h − RX ^t + T0h = 819.1 # 2
(C) 2RX ^t h + 2RX ^t − T0h Eb = 819.1 # 2
Rb
(D) 2RX ^t h - 2RX ^t - 2T0h = 31.503

S1 Correct option is (B) Q61 The bit error probability of a memoryless binary
Y ^ t h = X ^ t h − X ^t − T0h symmetric channel is 10-5 . If 105 bits are sent over this
ACf of o/p channel,then the probability that not more than one bit
Page 16 EC-Gate 2016 SET-3

will be in error is ______ S1 Correct option is (C)


a = 2.286 cm
S1 Correct answer is 0.735 b = 1.016 cm
P = 10−5 N air filled RWG
= 105 fc^TE h = c 1 + 1 (a m = 1, n = 1)
11
2 a2 b2
Method 1: 10
Binomial Method: nC x px qn - x = 3 # 10
1 1
c ^2.216h2 + ^1.016h2 m
P ^x = 0h + P 6x = 1@ = 105 c 0 ^10−5h0 ^1 − 10−5h10
5 2
fc^TE h = 16.15 GHz
+ 105 c1 ^10−5h1 ^1 − 10−5h10 − 1
5

11

= ^1 h^1 h # 0.367 + 0.367 fc^TE h = c


11
2b
= 0.735 10
= 3 # 10
Method 2: 201.016
−λ x
= 14.76 GHz
Poisson method = e λ
x! fc^TE h = c
λ = np = 10−5 # 105
20
a
10
=1 = 3 # 10
e−1 ]1g1 2.286
= + e−1 = 13.12 GHz
1!
& 2 # e-1 = 0.735 fc^TE h = c
20
2a
10

Q62 Consider an air-filled rectangular waveguide with


= 3 # 10
2 # 2.286
dimensions a = 2.286 cm and b = 1.016 cm. At 10 GHz = 6.56 GHz
operating frequency, the value of the propagation constant ` Increasing order of the cut-off frequency is given by
(per meter) of the corresponding propagating mode is TE 10 < TE 20 < TE 01 < TE 11
______
Q64 A radar operating at 5 GHz uses a common antenna
S1 Correct answer is 158.07 for transmission and reception. The antenna has a gain of
Given 150 and is aligned for maximum directional radiation and
Air filled RWG, reception to a regret 1 km away having radar cross-section
a = 2.286 cm of 3 m2 . If it transmit 100 kW, then the received power
b = 1.016 cm (in µW ) is
f = 10 GHz
Assume dominant mode ^TE 10h is propagating in the S1 Correct answer is 0.0122
waveguide, cut-off frequency of TE 10 mode is given by Given
fc ^TE 10h = c frequency, f = 5 GHz
2a
10 = 5 # 109 Hz
= 3 # 10 wave length, λ = c
2 # 2.286 f
fc = 6.56 GHz 8

Propagation constant γ is given by


= 3 # 109
5 # 10
γ = jβv
= 0.06 m
= iω µ 0 ε 0 1 − c c m
f 2
gain of antenna, G = 150
f
Range of target, R max = 1 km
1 1 − b 6.56 l
2
= j2π # 10 # 109 # = 103 m
3 # 10 8 10
γ = j158.07 m
−1 radar cross-section, σ = 3 m2
Therefore the value of propagation constant is given by transmitted power, Pt = 100 kW
γ = 158.07 m−1 The RADAR range equation is given by
2
P # G # 4λπ # G # σ
R max = > t H
^4π42 # PRh

ca Ae = λ G m
2
Q63 Consider an air-filled rectangular waveguide with

dimensions a = 2.286 cm and b = 1.016 cm. The increasing 4π
The received power, PR is given by
100 # 103 # 150 # 150 # ^0.06h2 # 3
order of the cut-off frequencies for different modes is
(A) TE 01 < TE 10 < TE 11 < TE 20 PR =
4π43 # ^103h4
(B) TE 20 < TE 11 < TE 10 < TE 01
= 1.22 # 10−8
(C) TE 10 < TE 20 < TE 01 < TE 11
= 0.0122 µW
(D) TE 10 < TE 11 < TE 20 < TE 01
EC-Gate 2016 SET-3 Page 17

Q65 Consider the charge profile shown in figure. The


resultant potential distribution is best described by

At t = 0+ :
^t − 1h2
y =
2
y = 1
2

S1 Correct answer is (D)


Let us consider b =− 1 and a = 1
For line (1):
Here (-1,0) to (0,-1) the line equation is
y - 0 = − 1 ^t + 1h
1
y =− t − 1
−^t + 1h2
−^t + 1h dt =
t
#−1 2
1<1<0
For line (2)
Here ^0, - 1h to (1, 0) the line equation is
y = t − 1
^t − 1h2
^t - 1h dt =
t
# 0 2
0<t<1

At t = 0− :
−^t + 1h2
y =
2
y = − 1
2

You might also like